View Single Post
  #273  
Old 06-11-2017, 04:18 PM
Newview01 Newview01 is offline
Banned
 
Join Date: Oct 2013
Posts: 5,326
Default

Quote:
Originally Posted by Big Thumper View Post
Having read your previous posts, I have no doubt you do not have the background to understand whether its mathematically possible or not. You've reached a conclusion without this knowledge as it fits what you are comfortable with.
Nobody has attempted to explain it...